المساعد الشخصي الرقمي

مشاهدة النسخة كاملة : متباينة 3


mathson
28-05-2008, 09:25 PM
إذا كان a,b,c أعداد حقيقية أكبر من الصفر فإن :


http://www.uaemath.com/ar/aforum/math0305447001211995546.png

محمودالجمال2007
28-05-2008, 10:23 PM
مسألة رائعة

mourad24000
28-05-2008, 11:09 PM
الله يخليك يا أخي mathson مسائل في القمة
بارك الله فيك
سأحاول في حل هذه المتباينة

prime
29-05-2008, 12:48 AM
عرف الدالة : f(x)=e^(2/3 x ln x ) -1 then we need to prove that

f(x)> 0
هذا مفتاح الحل استخدم المشتقه ستجد أن هذا صحيح لكل القيم بعد 1/e

والله العالم
اتمنى ان اكون لم اخطئ في الحسابات

mourad24000
29-05-2008, 03:30 AM
فكرة حل المتفاوتة هي استعمال الدالة اللوغارتمية (ln(x و بفرض a , b, c أعداد موجبة أكبر من 1

فيصبح البرهان على المتفاوتة التالية:http://www.uaemath.com/ar/aforum/math0211361001212017071.png
و بمأن الدالة( ln(x متزادة على مجموعة الأعداد الحقيقية الموجبة و الغير معدومة نحصل على المطلوب,
أرجوا أن تكون الفكرة صائبة
أين أستاذ ياسين موهوس المتفاوتات ليبدينا رأيه في طريقة الحل
سلامي الحار أخ mathson على نوعية معضلاتك الرائعة

mourad24000
29-05-2008, 03:37 AM
عذرا التكافئ الصحيح هو التالي:http://www.uaemath.com/ar/aforum/math0492600001212017842.png

ياسين
29-05-2008, 05:58 PM
اهلا اخ مراد لا يمكنني الرد لا ني لم ادرس ln فهو يدرس في السنة النهائية تانوي و اخوك مازال في ما قبل الاخيرة

mohamedegm
29-05-2008, 07:02 PM
السلام عليكم و رحمة الله.
سأكتب حلها قريبا.

mohamedegm
03-06-2008, 07:18 PM
السلام عليكم و رحمة الله.
لقد سبقتني إليها أخي Mathson كنت قد قررت أن أضعها في المنتدى يومها.

أما حلي لها فهو كالآتي:
==) الطريقة الأولى:
لدينا:
http://www.eclasshome.com/attach/upload3/wh_48613281.gif

و لدينا:
http://www.eclasshome.com/attach/upload3/wh_15368652.gif

بالتماثل (بين a ,b , c ) نعتبر أن: a>=b>=c

إدن:
http://www.eclasshome.com/attach/upload3/wh_41054687.gif

ومنه التكافؤ صحيح، وبالتالي النتيجة المطلوبة.

==) الطريقةالثانية :
تعتمد على متفاوتة الهارمونيكا
و أترك البرهان لمن أراد المحاولة.

و لكي لا أنسى فهده المتفاوتة كندية.

souzuki
04-06-2008, 03:44 PM
السلام عليكم ورحمة الله وبركاته

هذه المتراجحة وردت في أوليمبياد USA عام 1974

وفكرتها مبنية على استخدام Chebysev's inequality وكذلك فهم اللوغاريتمات

بدون المساس بعمومية المسألة نفرض أن

\Large a \leq b \leq c

وهذا يستلزم

\Large \ln a \leq \ln b \leq \ln c

لدينا الآن متتابعتين يمكن تطبيق Chebysev's inequality

\Large a \ln a+ b \ln b + c \ln c \geq \frac{(a + b + c)(\ln a + \ln b + \ln c)}{3}

\Large \ln a^a+ \ln b^b + \ln c^c \geq \frac{(a + b + c)}{3}(\ln a + \ln b + \ln c)

\Large \ln (a^a b^b c^c ) \geq \frac{(a + b + c)}{3}(\ln abc)

\Large \ln (a^a b^b c^c ) \geq \ln (abc)^{\frac{(a + b + c)}{3}}

\Large \Longrightarrow (a^a b^b c^c ) \geq (abc)^{\frac{(a + b + c)}{3}}

مع الشكر للجميع

jockereda
20-06-2008, 03:05 PM
شكرا لكم على تعليمنا كل يوم المزيد و المزيد..والسلام عليكم..

jawad soulaimani
12-05-2009, 02:09 AM
معدرة يا إخواني لكنكم إعتمدتم في براهينكم على حالة واحدة.فأين هي الحالات الأخرى؟لمادا بالضبط إخترتم حالة c<b<a!!!

mathson
12-05-2009, 11:07 AM
السلام عليكم ورحمة الله وبركاته

هذه المتراجحة وردت في أوليمبياد USA عام 1974

وفكرتها مبنية على استخدام Chebysev's inequality وكذلك فهم اللوغاريتمات

بدون المساس بعمومية المسألة نفرض أن

\Large a \leq b \leq c

وهذا يستلزم

\Large \ln a \leq \ln b \leq \ln c

لدينا الآن متتابعتين يمكن تطبيق Chebysev's inequality

\Large a \ln a+ b \ln b + c \ln c \geq \frac{(a + b + c)(\ln a + \ln b + \ln c)}{3}

\Large \ln a^a+ \ln b^b + \ln c^c \geq \frac{(a + b + c)}{3}(\ln a + \ln b + \ln c)

\Large \ln (a^a b^b c^c ) \geq \frac{(a + b + c)}{3}(\ln abc)

\Large \ln (a^a b^b c^c ) \geq \ln (abc)^{\frac{(a + b + c)}{3}}

\Large \Longrightarrow (a^a b^b c^c ) \geq (abc)^{\frac{(a + b + c)}{3}}

مع الشكر للجميع

هناك حل آخر باستخدام جينسن:

\Large \frac{a\ln a + b\ln b + c\ln c}{3} \ge \left(\frac{a+b+c}{3} \right ) \ln \left(\frac{a + b + c}{3} \right )\ge \left(\frac{a+b+c}{3} \right ) \ln \left(\sqrt[3]{abc} \right ) = \left(\frac{a+b+c}{9} \right ) \ln \left(abc \right )

ياسين
12-05-2009, 11:52 PM
هناك حل آخر باستخدام جينسن:

\Large \frac{\ln a + \ln b + \ln c}{3} \ge \left(\frac{a+b+c}{3} \right ) \ln \left(\frac{a + b + c}{3} \right )\ge \left(\frac{a+b+c}{3} \right ) \ln \left(\sqrt[3]{abc} \right ) = \left(\frac{a+b+c}{9} \right ) \ln \left(abc \right )


اعتقد انه هناك خطا في استعمالها ، حيت بدل a+b+c على 3 في الطرف التاني يجب لن يكون 1 ، او اني اغفلت مرحلة
ياليت توضح اخي

mathson
13-05-2009, 07:13 AM
اعتقد انه هناك خطا في استعمالها ، حيت بدل a+b+c على 3 في الطرف التاني يجب لن يكون 1 ، او اني اغفلت مرحلة
ياليت توضح اخي

بارك الله فيك، بالفعل هو خطأ، وتم التصحيح.